Phương pháp chuẩn hóa và phương pháp thuần nhất trong chứng minh bất đẳng thức

26 444 0
Phương pháp chuẩn hóa và phương pháp thuần nhất trong chứng minh bất đẳng thức

Đang tải... (xem toàn văn)

Tài liệu hạn chế xem trước, để xem đầy đủ mời bạn chọn Tải xuống

Thông tin tài liệu

Header Page of 133 BỘ GIÁO DỤC VÀ ĐÀO TẠO ĐẠI HỌC ĐÀ NẴNG TRẦN ÁNH NGỌC PHƯƠNG PHÁP CHUẨN HOÁ VÀ PHƯƠNG PHÁP THUẦN NHẤT TRONG CHỨNG MINH BẤT ĐẲNG THỨC Chuyên ngành : Phương pháp toán sơ cấp Mã số : 60.46.40 TÓM TẮT LUẬN VĂN THẠC SĨ KHOA HỌC Đà Nẵng − Năm 2011 Footer Page of 133 Header Page of 133 Công trình hoàn thành ĐẠI HỌC ĐÀ NẴNG Người hướng dẫn khoa học: TS Cao Văn Nuôi Phản biện 1: PGS.TSKH Trần Quốc Chiến Phản biện 2: GS.TSKH Nguyễn Văn Mậu Luận văn bảo vệ Hội đồng chấm Luận văn tốt nghiệp thạc sĩ Khoa học họp Đại học Đà Nẵng vào ngày 29 tháng năm 2011 Có thể tìm hiểu luận văn tại: − Trung tâm Thông tin − Học liệu, Đại học Đà Nẵng − Thư viện trường Đại học sư phạm, Đại học Đà Nẵng Footer Page of 133 Header Page of 133 Mở đầu LÝ DO CHỌN ĐỀ TÀI Trong chương trình toán học phổ thông bất đẳng thức nội dung khó cho người dạy lẫn người học Mặc khác phần lớn bất đẳng thức nên việc nghiên cứu phương pháp chứng minh bất đẳng thức phương pháp biến bất đẳng thức không nhất, có điều kiện dạng sau chứng minh nghiên cứu, giải lớp rộng toán bất đẳng thức Với mong muốn sử dụng nguồn kiến thức sở sơ cấp để giải lớp toán bất đẳng thức phục vụ thiết thực cho việc dạy học chương trình phổ thông, bồi dưỡng học sinh giỏi, chọn đề tài "Phương pháp chuẩn hoá phương pháp chứng minh bất đẳng thức" MỤC ĐÍCH NGHIÊN CỨU Mục đích đề tài trình bày có hệ thống từ sở lý thuyết hàm số đến bất đẳng thức Sau trình bày số phương pháp chứng minh bất đẳng thức nhất; phương pháp chuyển bất đẳng thức không nhất, có điều kiện dạng chứng minh Ngoài vận dụng lý thuyết để sáng tạo số bất đẳng thức ĐỐI TƯỢNG VÀ PHẠM VI NGHIÊN CỨU Đối tượng nghiên cứu Nghiên cứu bất đẳng thức nhất, bất đẳng thức đối xứng, bất đẳng thức AM-GM,bất đẳng thức Cauchy-Schwarz bất đẳng thức Schur, bất đẳng thức Muirhead ứng dụng chúng Phạm vi nghiên cứu Nghiên cứu lớp toán bất đẳng thức đại số chương trình toán phổ thông, kỳ thi học sinh giỏi quốc gia quốc tế PHƯƠNG PHÁP NGHIÊN CỨU Phương pháp nghiên cứu tư liệu, phương pháp thực nghiệm trường phổ thông phương pháp thảo luận trao đổi qua bạn bè, đồng nghiệp Ý NGHĨA KHOA HỌC VÀ THỰC TIỄN Tạo tài liệu bất đẳng thức để tham khảo nghiên cứu, giảng dạy Footer Page of 133 Header Page of 133 bồi dưỡng học sinh giỏi CẤU TRÚC CỦA LUẬN VĂN Ngoài phần mở đầu kết luận, luận văn gồm có chương: Chương Kiến thức Chương Bất đẳng thức Chương Phương pháp chuẩn hoá phương pháp chứng minh bất đẳng thức Footer Page of 133 Header Page of 133 Chương Kiến thức Hàm số Định nghĩa 1.1 Hàm số thực f (x1 , x2 , , xn ) biến số thực x1 , x2 , , xn xác định miền D gọi hàm số với (x1 , x2 , , xn ) ∈ D, với số thực t cho (tx1 , tx2 , , txn ) ∈ D tồn số thực m cho: f (tx1 , tx2 , , txn ) = tm f (x1 , x2 , , xn ) Khi ta nói hàm số f (x1 , x2 , , xn ) hàm số bậc m Đa thức đối xứng Định nghĩa 1.2 Hàm số f (x1 , x2 , , xn ) biến số thực x1 , x2 , , xn xác định D gọi hàm số đối xứng D : (x1 , x2 , , xn ) ∈ D (xσ(1) , xσ(2) , · · · , xσ(n) ) ∈ D f (x1 , x2 , , xn ) = f (xσ(1) , xσ(2) , · · · , xσ(n) ) σ hoán vị {1, 2, · · · , n.} Định nghĩa 1.3 Đa thức P (x1 , x2 , , xn ) biến số thực x1 , x2 , , xn gọi đa thức đối xứng hàm số đối xứng 2.1 Biểu diễn đa thức đối xứng qua đa thức đối xứng Định lý 1.1 Với số (n1 , n2 , n3 ) ∈ N3 thoả mãn n1 ≥ n2 ≥ n3 có hữu hạn số (t1 , t2 , t3 ) ∈ N3 cho: t1 ≥ t2 ≥ t3 vàn1 ≥ t1 , n1 + n2 ≥ t1 + t2 , n1 + n2 + n3 = t1 + t2 + t3 (Khi ta nói số (n1 , n2 , n3 ) trội số (t1 , t2 , t3 ) ký hiệu:(n1 , n2 , n3 ) (t1 , t2 , t3 ).) Định lý 1.2 Giả sử f (x1 , x2 , , xn ) ∈ R[x1 , x2 , , xn ] đa thức đối xứng khác đa thức không, có đa thức h(x1 , x2 , , xn ) ∈ R[x1 , x2 , , xn ] Footer Page of 133 Header Page of 133 cho f (x1 , x2 , , xn ) = h(p1 , p2 , , pn ) p1 , p2 , , pn đa thức đối xứng 2.2 Phương pháp biểu diễn đa thức đối xứng biến qua đa thức đối xứng Xét đa thức đối xứng ba biến P (x1 , x2 , x3 ), có hạng tử cao là: x1 α1 x2 α2 x3 α3 Đặt p = x1 + x2 + x3 , q = x1 x2 + x2 x3 + x1 x3 , r = x1 x2 x3 Khi ta tìm tất số (ti1 , ti2 , ti3 ) thoả mãn: (α1 , α2 , α3 ) (t11 t12 , t13 ) (t21 t22 , t23 ) (tm1 tm2 , tm3 ) sau ta biểu diễn: m P (x1 , x2 , x3 ) = λ0 p α1 −α2 α2 −α3 α3 q r λi pti1 −ti2 q ti2 −ti3 rti3 + i=1 Cuối dùng phương pháp đồng thức ( tức cho số (x1 , x2 , x3 ) nhận m + giá trị khác nhau) ta tìm tất hệ số λi , i = 0, 1, · · · , m suy biểu diễn P (x1 , x2 , x3 ) Bất đẳng thức Schur 3.1 Định lý Schur Với số thực không âm x, y, z, với số thực dương r ta có: xr (x − y)(x − z) + y r (y − z)(y − x) + z r (z − x)(z − y) ≥ Đẳng thức xảy khi: x = y = z x = y z = hoán vị chúng 3.2 Hệ Nếu x, y, z số thực dương bất đẳng thức với số thực r dấu đẳng thức xảy x = y = z Định nghĩa 1.4 (Tổng hoán vị vòng tròn tổng đối xứng ) : Cho hàm số f (x, y, z) ba biến số thực x, y, z , đó: i> Ta gọi tổng hoán vị vòng tròn biến hàm số f (x, y, z) ký hiệu xác định là: f (x, y, z) = f (x, y, z) + f (y, z, x) + f (z, x, y) cyclic Footer Page of 133 Header Page of 133 ii> Ta gọi tổng đối xứng biến hàm số f (x, y, z) ký hiệu xác định là: f (x, y, z) = f (x, y, z)+f (x, z, y)+f (y, x, z)+f (y, z, x)+f (z, x, y)+f (z, y, x) sym 3.3 Bất đẳng thức Schur suy rộng Bộ số thực (x, y, z) gọi số đơn điệu ta có: x ≥ y ≥ z z ≥ y ≥ x Định lý 1.3 Với số thực không âm: a, b, c, x, y, z cho số (a, b, c), (x, y, z) số đơn điệu ta có x(a−b)(a−c)+y(b−c)(b−a)+z(c−a)(c−b) ≥ Đẳng thức xảy khi: (a = b = c) ( a = b z = ) ( b = c x = ) 4.1 Định lý Muirhead Khái niệm làm trội Cho hai số thực: a = (a1 , a2 , · · · , an ), a = (a1 , a2 , · · · , an ), ta nói số a’ bị làm trội số a hay số a trội số a’ nếu: a1 ≥ a2 ≥ · · · ≥ an ; a1 ≥ a2 ≥ · · · ≥ an , a1 + a2 + · · · + ak ≥ a1 + a2 + · · · + ak , ≤ k < n, a1 + a2 + · · · + an = a1 + a2 + · · · + an (a1 , a2 , , an ) (a1 , a2 , , an ) ≺ (a1 , a2 , , an ) Khi ta ký hiệu: (a1 , a2 , , an ) 4.2 Định nghĩa phép biến đổi T Cho số thực α = (α1 , α2 , · · · , αn ), giả sử αk αl hai số khác l l α αk > αl Đặt: p = αk +α , t = αk −α , ta có: 2 t > 0, αk = p + t, αl = p − t Khi với số thực q cố định thoả mãn: ≤ q < t ta xác định phép biến đổi T sau: αk =p+q αl =p−q α i = αi , i = k, i = l T (α) = α ⇔ Nếu α = T (α) ta nói α thu từ α nhờ phép biến đổi T hay T biến đổi α thành α Bây cho số dương (x1 , x2 , · · · , xn ), ký hiệu: x1 α1 x2 α2 · · · xn αn tổng đối sym α1 xứng biểu thức dạng: x1 x2 Footer Page of 133 α2 · · · xn αn ta có bổ đề sau: Header Page of 133 x1 α x2 α · · · xn α n ≤ Bổ đề 1.1 Nếu α = T (α) sym x1 α1 x2 α2 · · · xn αn sym Đẳng thức xảy khi: x1 = x2 = · · · = xn Bổ đề 1.2 Nếu α ≺ α α không đồng với α tồn α∗ cho α ≺ α∗ α∗ thu từ α nhờ thực liên tiếp số hữu hạn phép biến đổi T Bổ đề 1.3 Nếu α ≺ α α thu từ α nhờ áp dụng liên tiếp hữu hạn lần phép biến đổi T 4.3 Định lý Muirhead tổng quát Cho n số dương x1 , x2 , · · · , xn , cho hai số không âm α = (α1 , α2 , · · · , αn ) α = (α , α , · · · , α n ) thoả điều kiện: α1 ≥ α2 ≥ · · · ≥ αn ≥ 0; α ≥ α ≥ · · · ≥ α n ≥ Khi điều kiện cần đủ để x1 α x2 α · · · xn α n α x1 α1 x2 α2 · · · xn αn ≥ sym α sym Đẳng thức xảy khi: αi = α i i = 1, n x1 = x2 = · · · = xn 4.4 Định lý Muirhead cho số Cho số thực a1 , a2 , a3 , b1 , b2 , b3 thoả: a1 ≥ a2 ≥ a3 ≥ 0; b1 ≥ b2 ≥ b3 ≥ 0; a1 ≥ b1 , a1 + a2 ≥ b1 + b2 ; a1 + a2 + a3 = b1 + b2 + b3 xb1 y b2 z b3 xa1 y a2 z a3 ≥ Cho x, y, z số thực dương, sym sym Đẳng thức xảy khi: = bi i = 1, 2, x = y = z Để chứng minh định lý trường hợp ta cần đến bổ đề sau: Bổ đề 1.4 Cho số thực không âm a1 , a2 , b1 , b2 thoả: a1 + a2 = b1 + b2 max{a1 ; a2 } ≥ max{b1 ; b2 } Khi với số thực dương x, y ta có : xa1 y a2 + xa2 y a1 ≥ xb1 y b2 + xb2 y b1 Đẳng thức xảy khi: a1 = b1 a2 = b2 x = y Footer Page of 133 Header Page of 133 Bất đẳng thức trung bình cộng trung bình nhân (Bất đẳng thức AM - GM) Định lý 1.4 Với n số thực không âm bất kỳ: a1 , a2 , · · · , an , (1 < n ∈ N), đặt AM = Ta có: a1 + a2 + · · · + an , n GM = √ n a1 a2 · · · an AM ≥ GM Đẳng thức xảy khi: a1 = a2 = · · · = an Bất đẳng thức Cauchy-Schwarz Định lý 1.5 Với n số thực (a1 , a2 , , an ), (b1 , b2 , , bn ) ta có: b i ) ≤ ( )( i=1 i=1 bi ) 2 ( n n n i=1 Đẳng thức xảy khi: a1 a2 an = = ··· = b1 b2 bn Footer Page of 133 Header Page 10 of 133 Chương Bất đẳng thức Bất đẳng thức Định nghĩa 2.1 Bất đẳng thức dạng f (x1 , x2 , , xn ) ≥ f (x1 , x2 , , xn ) hàm số gọi bất đẳng thức Khi bất đẳng thức: f (x1 , x2 , , xn ) ≤ 0; f (x1 , x2 , , xn ) > 0; f (x1 , x2 , , xn ) < gọi bất đẳng thức 2.1 Một số phương pháp chứng minh bất đẳng thức Phương pháp dồn biến Để chứng minh bất đẳng thức f (x1 , x2 , , xn ) ≥ trước hết ta chứng minh √ f (x1 , x2 , , xn ) ≥ f (t, t, x3 , , xn ), đó: t = x1 +x t = x1 x2 Để chứng minh f (x1 , x2 , , xn ) ≥ f (t, t, x3 , , xn ) ta thường chứng minh: f (x1 , x2 , , xn ) − f (t, t, x3 , , xn ) ≥ Cuối để kết thúc việc chứng minh ta cần chứng minh f (t, t, x3 , , xn ) ≥ bất đẳng thức có số biến bất đẳng thức ban đầu Trong trình chứng minh ta dùng thêm số biện pháp sau: i> Nếu bất đẳng thức đối xứng với biến ta giả sử: x1 ≤ x2 ≤ ≤ xn x1 ≥ x2 ≥ ≥ xn ii> Nếu bất đẳng thức có dạng hoán vị vòng quanh ta giả sử: x1 = min{x1 , x2 , , xn } x1 = max{x1 , x2 , , xn } Hiển nhiên thủ thuật không làm giảm tính tổng quát toán ban đầu Minh hoạ phương pháp: Bài toán 2.1 ( Czech- Slovakia 1999 ) Cho số thực dương a,b,c Chứng minh rằng: b c a + + ≥1 b + 2c c + 2a a + 2b Footer Page 10 of 133 10 Header Page 12 of 133 Lời giải: Bất đẳng thức cần chứng minh tương đương với: a(b + c) b(c + a) c(a + b) 8a2 b2 c2 + + − − ( − 1) ≥ b2 + c2 c2 + a2 a2 + b (a2 + b2 )(b2 + c2 )(c2 + a2 ) Ta có: b(a − b) c(a − c) b(c + a) a(b + c) − = + ; −1= b2 + c2 (b2 + c2 ) (b2 + c2 ) c2 + a2 c(a + b) −1= a2 + b c(b − c) a(b − a) + ; (c2 + a2 ) (c2 + a2 ) a(c − a) b(c − b) + (a2 + b2 ) (a2 + b2 ) Lại có: b(a − b) a(b − a) (a − b)2 (ab − c2 ) c(b − c) b(c − b) (b − c)2 (bc − a2 ) + = ; + = ; (b2 + c2 ) (c2 + a2 ) (b2 + c2 )(c2 + a2 ) (c2 + a2 ) (a2 + b2 ) (c2 + a2 )(a2 + b2 ) c(a − c) (c − a)2 (ca − b2 ) a(c − a) + = (a2 + b2 ) (b2 + c2 ) (a2 + b2 )(b2 + c2 ) Và: 8a2 b2 c2 (a2 + b2 )(b2 + c2 )(c2 + a2 ) − 8a2 b2 c2 −1=− (a2 + b2 )(b2 + c2 )(c2 + a2 ) (a2 + b2 )(b2 + c2 )(c2 + a2 ) a2 (b2 − c2 )2 + b2 (c2 − a2 )2 + c2 (a2 − b2 )2 =− (a2 + b2 )(b2 + c2 )(c2 + a2 ) a2 (b + c)2 (b − c)2 + b2 ((c + a)2 (c − a)2 + c2 (a + b)2 (a − b)2 =− (a2 + b2 )(b2 + c2 )(c2 + a2 ) Do bất đẳng thức cần chứng minh đưa dạng tương đương là: Sa (b − c)2 + Sb (c − a)2 + Sc (a − b)2 ≥ Trong đó: Sa = bc − a2 a2 (b + c)2 bc(2a2 + b2 + c2 ) + = ≥ 0; (a2 + b2 )(c2 + a2 ) (a2 + b2 )(b2 + c2 )(c2 + a2 ) (a2 + b2 )(b2 + c2 )(c2 + a2 ) ca(2b2 + c2 + a2 ) ab(2c2 + a2 + b2 ) Sb = ≥ 0; Sc = ≥ (a + b2 )(b2 + c2 )(c2 + a2 ) (a + b2 )(b2 + c2 )(c2 + a2 ) Nên theo i> bất đẳng thức: Sa (b − c)2 + Sb (c − a)2 + Sc (a − b)2 ≥ Vậy bất đẳng thức ban đầu chứng minh 2.3 Phương pháp ứng dụng bất đẳng thức Schur bất đẳng thức Schur suy rộng Định lí Schur: Với số thực không âm: x, y, z, với số thực dương r ta có: xr (x − y)(x − z) + y r (y − z)(y − x) + z r (z − x)(z − y) ≥ Footer Page 12 of 133 11 Header Page 13 of 133 Đẳng thức xảy khi: x = y = z x = y z = hoán vị chúng Minh hoạ phương pháp: Bài toán 2.3 (Balkan Mathematical Olympiad) Cho số thực dương a, b, c Chứng minh rằng: a3 b3 c3 3(ab + bc + ca) + + ≥ b2 − bc + c2 c2 − ca + a2 a2 − ab + b2 a+b+c Lời giải: Ta có: a+b+c≥ 3(ab + bc + ca) ⇔ (a − b)2 + (b − c)2 + (c − a)2 ≥ a+b+c Nên ta cần chứng minh: a3 b3 c3 + + ≥a+b+c b2 − bc + c2 c2 − ca + a2 a2 − ab + b2 Theo bất đẳng thức Cauchy-Schwarz ta có: √ a a √ ( a) =[ b2 − bc + c2 cyclic cyclic 2 ≤ a(b2 − bc + c2 )]2 a3 a(b2 − bc + c2 ) 2 b − bc + c cyclic cyclic hay: a ≥ − bc + c2 b cyclic a(b2 − bc + c2 ) = Ta có: cyclic a(b2 − bc + c2 ) cyclic a2 b − sym cyclic a2 )2 ( abc nên: cyclic a(b2 − bc + c2 ) = (a + b + c)( (a + b + c) sym cyclic a3 b + = sym cyclic ( a2 bc − sym a2 )2 = ( Và: a2 b2 + sym a4 + cyclic Footer Page 13 of 133 a2 bc = cyclic cyclic a3 b + sym abc) a2 b − sym a2 bc cyclic a2 b2 Do đó: sym a2 )2 ≥ (a + b + c) cyclic a2 b − a(b2 − bc + c2 ) ⇔ cyclic a4 + cyclic a2 bc ≥ cyclic a3 b sym 12 Header Page 14 of 133 theo bất đẳng thức Schur với r = Suy ra: a2 )2 ( cyclic a(b2 − bc + c2 ) ≥a+b+c cyclic Tóm lại ta có: ( a ≥ − bc + c2 b cyclic a2 )2 cyclic a(b2 − bc + c2 ) ≥a+b+c≥ 3(ab + bc + ca) a+b+c cyclic Vậy bất đẳng thức chứng minh Bất đẳng thức Schur suy rộng: Với số thực không âm: a, b, c, x, y, z cho số (a, b, c), (x, y, z) số đơn điệu ta có: x(a−b)(a−c)+y(b−c)(b−a)+z(c−a)(c−b) ≥ Đẳng thức xảy khi: a = b = c a = b z = b = c x=0 Minh hoạ phương pháp: Bài toán 2.4 Cho số thực dương a, b, c Chứng minh rằng: a2 + bc b2 + ac c2 + ab a b c + + ≥ + + 3 3 2 b +c c +a a +b b +c c +a a + b2 Lời giải: Vì bất đẳng thức có tính đối xứng nên không giảm tổng quát, giả sử: a ≥ b ≥ c Ta có: a a(b + c) a a ≥ ⇒ ≥ b2 + c2 − bc b2 + c2 b3 + c3 b + c2 Tương tự: b(c + a) b c(a + b) c ≥ , ≥ c3 + a3 c2 + a2 a3 + b3 a2 + b Ta chứng minh bất đẳng thức mạnh là: a2 + bc b2 + ac c2 + ab a(b + c) b(c + a) c(a + b) + + ≥ + + b3 + c3 c + a3 a3 + b3 b + c3 c + a3 a + b3 a2 + bc − ab − ac b2 + ac − bc − ba c2 + ab − ca − cb ⇔ + + ≥0 b3 + c3 c3 + a3 a3 + b 1 ⇔ (a − b)(a − c) + (b − c)(b − a) + (c − a)(c − b) ≥ (2.1) 3 b +c c +a a + b3 Đặt: x= Footer Page 14 of 133 b3 1 ,y = ,z = 3 +c c +a a + b3 13 Header Page 15 of 133 Vì a ≥ b ≥ c nên a3 + b3 ≥ a3 + c3 ≥ b3 + c3 suy x ≥ y ≥ z Khi đó: (2.1) ⇔ x(a − b)(a − c) + y(b − c)(b − a) + z(c − a)(c − b) ≥ theo bất đẳng thức Schur suy rộng Vậy bất đẳng thức chứng minh Phương pháp ứng dụng định lí Muirhead 2.4 Định lí Muirhead cho số: Cho số thực a1 , a2 , a3 , b1 , b2 , b3 thoả: a1 ≥ a2 ≥ a3 ≥ 0; b1 ≥ b2 ≥ b3 ≥ 0; a1 ≥ b1 , a1 + a2 ≥ b1 + b2 ; a1 + a2 + a3 = b1 + b2 + b3 xa1 y a2 z a3 ≥ Khi với số thực dương x, y, z ta có: sym x b1 y b z b3 sym Đẳng thức xảy khi: = bi i = 1, 2, x = y = z Minh hoạ phương pháp: Bài toán 2.5 (USAMO 1997) Cho số thực dương a, b, c Chứng minh rằng: 1 1 + + ≤ a3 + b3 + abc b3 + c3 + abc c3 + a3 + abc abc Lời giải: Bất đẳng thức cần chứng minh tương đương với: [(a3 + b3 + abc)(a3 + c3 + abc)abc] ≤ (a3 + b3 + abc)(b3 + c3 + abc)(c3 + a3 + abc) cyclic ⇔ (a7 bc+4a5 b2 c2 +3a4 b4 c+a3 b3 c3 ) ≤ sym a6 b ≥ ⇔ sym a5 b2 c2 (a7 bc+2a6 b3 +2a5 b2 c2 +3a4 b4 c+a3 b3 c3 ) sym theo Muirhead vì:(6; 3; 0) (5; 2; 2) sym Vậy bất đẳng thức chứng minh 2.5 Phương pháp ứng dụng tính chất đa thức đối xứng bậc Nếu P (u, v, w) đa thức đối xứng bậc R P (1, 0, 0) ≥ 0, P (1, 1, 0) ≥ 0, P (1, 1, 1) ≥ P (x, y, z) ≥ x, y, z ≥ Định lý 2.2 Footer Page 15 of 133 14 Header Page 16 of 133 Minh hoạ phương pháp: Bài toán 2.6 Cho số thực dương a, b, c Chứng minh rằng: a2 + b2 − ab b2 + c2 − bc c2 + a2 − ca + + ≥ (c + a)(c + b) (a + b)(a + c) (b + c)(b + a) Lời giải: Bất đẳng thức cần chứng minh tương đương với: a3 + b b + c3 c3 + a3 + + ≥ (a + b)(b + c)(c + a) (a + b)(b + c)(c + a) (a + b)(b + c)(c + a) ⇔ 2(a3 + b3 + c3 ) ≥ ⇔ 8(a3 + b3 + c3 ) − 3(a + b)(b + c)(c + a) ≥ (a + b)(b + c)(c + a) Xét: P (x, y, z) = 8(x3 + y + z ) − 3(x + y)(y + z)(z + x) ta có P (x, y, z) đa thức đối xứng bậc P (1, 0, 0) = 8, P (1, 1, 0) = 10, P (1, 1, 1) = Suy ra: P (a, b, c) ≥ a, b, c > Hay 8(a3 + b3 + c3 ) − 3(a + b)(b + c)(c + a) ≥ Vậy bất đẳng thức chứng minh Footer Page 16 of 133 15 Header Page 17 of 133 Chương Phương pháp chuẩn hoá phương pháp chứng minh bất đẳng thức Phương pháp chuẩn hoá Mệnh đề 3.1 Nếu f hàm n biến thực bậc m với t ∈ R, t > u ∈ Rn ta có f (u) ≥ tương đương với f (tu) ≥ Chứng minh: Vì hàm số f bậc m nên f (tu) = tm f (u) Do với t > ta có: f (tu) ≥ ⇔ tm f (u) ≥ ⇔ f (u) ≥ Mệnh đề chứng minh Nhận xét 3.1 Do mệnh đề nói nên để chứng minh bất đẳng thức dạng f (u) ≥ ta cần chứng minh bất đẳng thức f (tu) ≥ với tu thoả mãn điều kiện đặc biệt t > đó, việc chọn số t thích hợp giúp ta giảm bớt vất vả việc biến đổi biểu thức Sử dụng mệnh đề nói ta thu hẹp phạm vi cần xét biến so với yêu cầu toán Việc chuyển toán chứng minh bất đẳng thức việc chứng minh bất đẳng thức phạm vi hẹp biến gọi chuẩn hoá bất đẳng thức 1.1 Chứng minh bất đẳng thức phương pháp chuẩn hoá Bài toán 3.1 (Sưu tầm) Cho số thực dương a, b, c, chứng minh rằng: a2 b2 c2 a+b+c + + ≥ b+c c+a a+b Lời giải: Footer Page 17 of 133 16 Header Page 18 of 133 Đặt a2 b2 c2 a+b+c f (a, b, c) = + + − b+c c+a a+b f (a, b, c) hàm số bậc nên tồn t > mà f (ta, tb, tc) ≥ f (a, b, c) ≥ Với a, b, c > chọn t số thoả mãn ta + tb + tc = (hay t = a+b+c ) đặt a = ta, b = tb, c = tc Khi a , b , c > a + b + c = Ta cần chứng minh: f (a , b , c ) ≥ với a + b + c = a , b , c > (3.1) Do ta cần chứng minh: a2 b2 c2 + + ≥ 1−a 1−b 1−c (3.2) Theo bất đẳng thức Cauchy-Schwarz ta có: √ √ √ a b c + 1−b √ ) 12 = (a + b + c )2 = ( − a √ + 1−c √ 1−a 1−c 1−b a2 b2 c2 a2 b2 c2 ≤ (3 − a − b − c )( + + ) = 2( + + ) 1−a 1−b 1−c 1−a 1−b 1−c hay: a2 b2 c2 + + ≥ 1−a 1−b 1−c Bất đẳng thức (3.2) chứng minh suy bất đẳng thức cho chứng minh Nhận xét 3.2 Nếu không nhầm lẫn cách hiểu tránh phức tạp trình bày lời giải lời giải trên, kể từ dòng có ký hiệu (3.1) đến hết lời giải, ta dùng số (a, b, c) thay cho số (a , b , c ) Hay nói cách khác, với bất đẳng thức cho ta cần chứng minh trường hợp a, b, c thoả mãn điều kiện đặc biệt Từ toán 3.2 đến toán 3.4 ta sử dụng cách nói Bài toán 3.2 (Carlson) Cho số thực dương a, b, c, chứng minh rằng: (a + b)(b + c)(c + a) ≥ ab + bc + ca Lời giải: Bất đẳng thức cho thuộc dạng bất đẳng thức nên ta cần chứng minh trường hợp: ab + bc + ca = (ứng với t = ,) ta cần ab+bc+ca chứng minh : (a + b)(b + c)(c + a) ≥ Footer Page 18 of 133 17 Header Page 19 of 133 Ta có: (a + b + c)2 ≥ 3(ab + bc + ca) = ⇒ a + b + c ≥ Theo bất đẳng thức AM-GM: (abc)2 ≤ ab+bc+ca ⇒ (abc)2 ≤ ⇒ abc ≤ đó: (a+b)(b+c)(c+a) = (a+b+c)(ab+bc+ca)−abc = 3(a+b+c)−abc ≥ 3.3−1 = Vậy bất đẳng thức chứng minh Cho số thực dương a, b, c, chứng minh rằng: Bài toán 3.3 ( √ 1 + b + c6 ) ≥ + + )( a 3(a + b + c) a2 b c Lời giải: Bất đẳng thức cho thuộc dạng bất đẳng thức nên ta cần chứng minh trường hợp: a6 + b6 + c6 = (ứng với t = a6 +b36 +c6 ) ta cần chứng minh : 1 + + ≥a+b+c a2 b2 c2 hay: a2 b2 + b2 c2 + c2 a2 ≥ a2 b2 c2 (a + b + c) Theo bất đẳng thức AM-GM ta có: a2 b2 + b2 c2 ≥ 2b2 ac, b2 c2 + c2 a2 ≥ 2c2 ab, c2 a2 + a2 b2 ≥ 2a2 bc và: a6 + b + c 0 xyz = Khi bất đẳng thức cần chứng minh trở thành: ≥ x9 (y + z ) cyclic x4 y ≥ z (x3 + y ) cyclic hay Ta có: x4 y ≥ ⇔ x9 y (y +z )(z +x3 ) ≥ 3x5 y z (x3 +y )(y +z )(z +x3 ) 3 z (x + y ) cyclic cyclic x12 y 12 + ⇔2 x12 y z + sym cyclic x12 y 12 + ⇔ sym x12 y z + sym sym x11 y z + sym sym theo Muirhead Vậy bất đẳng thức chứng minh x8 y z ) ≥ sym x8 y z sym x12 y z − x9 y z − x8 y z sym sym x11 y z ) + 2( +( Footer Page 21 of 133 x9 y z ≥ sym x12 y 12 − x11 y z + sym cyclic sym ⇔( x9 y z ≥ x11 y z )+ sym 20 Header Page 22 of 133 Bài toán 3.7 Cho số thực dương a, b, c thoả mãn điều kiện ab+bc+ca = Chứng minh rằng: 1 + + ≥ (a + b)2 (b + c)2 (c + a)2 Lời giải: Sử dụng điều kiện: ab + bc + ca = để hoá, bất đẳng thức cần chứng minh trở thành: (ab + bc + ca)[ 1 + + ] ≥ (a + b)2 (b + c)2 (c + a)2 Đây dạng bất đẳng thức Iran 1996 Đặt: p = a + b + c, q = ab + bc + ca, r = abc p, q, r > và: (a + b)2 (a + c)2 1 p4 − 2p2 q + q + 4pr cyclic + + = = (a + b)2 (b + c)2 (c + a)2 [(a + b)(b + c)(c + a)]2 (pq − r)2 Do bất đẳng thức cần chứng minh tương đương với: 4(p4 q − 2p2 q + q + 4pqr) ≥ 9(p2 q + r2 − 2pqr) hay 4p4 q − 17p2 q + 34pqr + 4q − 9r2 ≥ hay pq(p3 −4pq+9r)+3q(p4 −5p2 q+4q +6pr)+2q(p2 q+3pr−4q )+r(pq−9r) ≥ Bất đẳng thức cuối theo (11’),(12’),(6’) (2’) Vậy bất đẳng thức chứng minh 3.1 Sáng tạo bất đẳng thức Sáng tạo bất đẳng thức từ bất đẳng thức Schur Từ bất đẳng thức Schur ta có: xr (x − y)(x − z) + y r (y − z)(y − x) + z r (z − x)(z − y) ≥ Với x, y, z ≥ 0, r > x, y, z > 0, r ∈ R Khi chọn r = 1, x, y, z > đặt x = a1 , y = 1b , z = + ≥ a abc cyclic Nhân hai vế với abc ta được: cyclic hay: cyclic a2 +b2 ab − cyclic ab c2 ≤3 ab c2 +3≥ hay: sym c ta có: a2 b a b sym a2 b2 +a2 c2 −b2 c2 a2 bc cyclic ≤3 Ta có toán sau: Bài toán thứ nhất: Cho số thực dương a, b, c chứng minh rằng: a2 b2 + a2 c2 − b2 c2 b2 c2 + b2 a2 − c2 a2 c2 a2 + c2 b2 − a2 b2 + + ≤ 3abc a b c Footer Page 22 of 133 21 Header Page 23 of 133 Từ bất đẳng thức Schur, đặt x = a1 , y = 1b , z = c ta có: −1 (b − c)(b − a) 1 1 ( − )( − ) = ar a b b c abc bar Xét tương tự với y r (y − z)(y − x) z r (z − x)(z − y) vào bất đẳng thức xr (x − y)(x − z) = Schur ta có toán sau: Bài toán thứ hai: Cho số thực dương a, b, c chứng minh với số thực x ta có: (a − b)(a − c)cx (b − c)(b − a)ax (c − a)(c − b)bx + + ≤0 a b c Từ bất đẳng thức Schur, đặt x = a + b, y = b + c, z = c + a ta có: xr (x − y)(x − z) ≥ ⇔ cyclic (a + b)r (a − b)(b − c) ≥ cyclic Chia hai vế cho [(a + b)(b + c)(c + a)]r ta có: cyclic (a−b)(a−c) [(a+b)(a+c)]r ≥0 Ta có toán sau: Bài toán thứ ba: Cho số thực không âm a, b, c có nhiều số Chứng minh với số thực dương x ta có: (a − b)(a − c) (b − c)(b − a) (c − a)(c − b) + + ≥0 x x [(a + b)(a + c)] [(b + c)(b + a)] [(c + a)(c + b)]x Từ bất đẳng thức Schur, đặt x = 2a + b, y = 2b + c, z = 2c + a ta có: xr (x − y)(x − z) ≥ ⇔ cyclic (2b + c)r (2a − b − c)(2b − c − a) ≤ cyclic Chia hai vế cho [(2a + b)(2b + c)(2c + a)]r ta có: (2a − b − c)(2b − c − a) ≤0 r [(2a + b)(2c + a)] cyclic Ta có toán sau: Bài toán thứ tư: Cho số thực không âm a, b, c có nhiều số Chứng minh với số thực dương r ta có: (2a − b − c)(2b − c − a) (2b − c − a)(2c − a − b) (2c − a − b)(2a − b − c) + + ≤0 [(2a + b)(2c + a)]r [(2b + c)(2a + b)]r [(2c + a)(2b + c)]r 3.2 Sáng tạo bất đẳng thức từ bất đẳng thức Schur suy rộng Ta biết bất đẳng thức Schur suy rộng: Với số thực không âm: a, b, c, x, y, z cho số (a, b, c), (x, y, z) số đơn điệu ta có: x(a − b)(a − c) + y(b − c)(b − a) + z(c − a)(c − b) ≥ Footer Page 23 of 133 22 Header Page 24 of 133 Đẳng thức xảy khi: a = b = c a = b z = b = c x=0 Trước hết giả sử a ≥ b ≥ c > ta có: ab ≥ ac ≥ bc, a + b ≥ a + c ≥ b + c, 1 ≥ ≥ , c b a 1 ≥ ≥ , bc ca ab 1 ≥ ≥ b+c c+a a+b Khi với hai số thực không âm m, n ta có: am bm cm ≥ ≥ (b + c)n (a + c)n (a + b)m Do ta có toán sau: Bài toán thứ năm: Cho số thực dương a, b, c chứng minh với số thực không âm m, n ta có: am (a − b)(a − c) bm (b − c)(b − a) cm (c − a)(c − b) + + ≥0 (b + c)n (c + a)n (a + b)n Nếu chọn m = n = bất đẳng thức trở thành: a3 + abc ≥ a2 + b + c b+c cyclic Khi xét thêm a2 + ≥ 2a, b2 + ≥ 2b, c2 + ≥ 2c hay: a2 + b2 + c2 ≥ 2(a + b + c) − ta có toán sau: Bài toán thứ sáu: Cho số thực dương a, b, c chứng minh rằng: a3 + abc b3 + abc c3 + abc + + ≥ 2(a + b + c) − b+c c+a a+b 3.3 Sáng tạo bất đẳng thức cách sử dụng phương pháp chuẩn hoá x2 (x − y)(x − z) ≥ Từ bất đẳng thức Schur: cyclic 2 xy − x + hay sym sym xy≥0 hay p4 + 3pr ≥ 5q sym Trong p = x + y + z, q = xy + yz + zx, r = xyz Lần lượt thực chuẩn hoá: q = 5, p = 5, pr = Bài toán thứ bảy: ta có toán sau: Cho số thực không âm x, y, z thoả mãn điều kiện xy + yz + zx = 5, chứng minh rằng: (x + y + z)4 + 3xyz(x + y + z) ≥ 125 Footer Page 24 of 133 23 Header Page 25 of 133 Bài toán thứ tám: Cho số thực không âm x, y, z thoả mãn điều kiện x + y + z = 5, chứng minh rằng: (xy + yz + zx)2 − 3xyz ≤ 125 Bài toán thứ chín: Cho số thực không âm x, y, z thoả mãn điều kiện x2 yz + y zx + z xy = 13 , chứng minh rằng: (x + y + z)4 − 5(xy + yz + zx)2 + ≥ Xét (x − y)(x3 − y ), B = A= x2 (x − y)(x − z) cyclic cyclic Ta có: x4 − mA + nB = m( sym x3 y) + n( sym x4 + cyclic x4 − (m + n) = (2m + n) cyclic x2 yz − sym sym cyclic x3 y + n x3 y) x2 yz cyclic = (2m + n)(p4 − 4p2 q + 2q + 4pr) − (m + n)(p2 q − 2q − pr) + npr = (2m + n)p4 − (9m + 5n)p2 q − 2(3m + 2n)q + 3(3m + 2n)pr Trong p = x + y + z, q = xy + yz + zx, r = xyz Với x, y, z, m, n ≥ mA + nB ≥ từ ta có kết tổng quát là: (2m + n)p4 − (9m + 5n)p2 q − 2(3m + 2n)q + 3(3m + 2n)pr ≥ Chọn n = 2m thực chuẩn hoá p = ta có toán sau: Bài toán thứ mười: Cho số thực dương x, y, z thoả mãn điều kiện x + y + z = 1, chứng minh rằng: 14(xy + yz + zx)2 − 19(xy + yz + zx) + 21xyz + ≥ Footer Page 25 of 133 24 Header Page 26 of 133 Kết luận Qua trình nghiên cứu hoàn thành luận văn này, tác giả thực số nội dung sau đây: Tìm tòi, lọc xếp theo trình tự, có hệ thống kiến thức gồm: Hàm số nhất; Hàm số đối xứng; Bất đẳng thức Schur; Bất đẳng thức Muirhead; Bất đẳng thức trung bình cộng trung bình nhân; Bất đẳng thức Cauchy- Schwarz Trình bày số phương pháp chứng minh bất đẳng thức gồm: Phương pháp dồn biến; Phương pháp phân tích bình phương; Phương pháp ứng dụng bất đẳng thức Schur; Phương pháp ứng dụng bất đẳng thức Muirhead ; Phương pháp ứng dụng tính chất đa thức đối xứng bậc ba phương pháp chuẩn hoá Trình bày phương pháp để chứng minh số toán bất đẳng thức có điều kiện Với phương pháp nêu có phần minh hoạ phương pháp qua việc chứng minh số bất đẳng thức đề thi học sinh giỏi quốc gia, quốc tế Vận dụng phối hợp bất đẳng thức Schur, Muirhead, bất đẳng thức trung bình cộng trung bình nhân phương pháp chuẩn hoá để đưa mười toán bất đẳng thức Với việc sử dụng kiến thức bản, phổ biến, sơ cấp, luận văn tài liệu tham khảo việc dạy học nội dung bất đẳng thức Footer Page 26 of 133 ... số phương pháp chứng minh bất đẳng thức gồm: Phương pháp dồn biến; Phương pháp phân tích bình phương; Phương pháp ứng dụng bất đẳng thức Schur; Phương pháp ứng dụng bất đẳng thức Muirhead ; Phương. .. thức đối xứng bậc ba phương pháp chuẩn hoá Trình bày phương pháp để chứng minh số toán bất đẳng thức có điều kiện Với phương pháp nêu có phần minh hoạ phương pháp qua việc chứng minh số bất đẳng... + b + c) ≥ a + b + c ≥ Vậy bất đẳng thức chứng minh 2.1 Phương pháp Thuần hoá bất đẳng thức không Trong phần trên, chứng minh số bất đẳng thức phương pháp khác Nhưng với bất đẳng thức không sao?

Ngày đăng: 19/05/2017, 09:28

Từ khóa liên quan

Tài liệu cùng người dùng

Tài liệu liên quan